Finding k such that u and v form {\pi}/3 Radians

  • Thread starter Thread starter phosgene
  • Start date Start date
  • Tags Tags
    Form Radians
phosgene
Messages
145
Reaction score
1

Homework Statement



Let u = (1,k) and v = (3,4). Find k such that the angle between u and v is {\pi}/3 radians.

Homework Equations



u{\bullet}v=||u|| ||v|| cos{\theta}=x_{1}x_{2}+y_{1}y_{2}

||u||=\sqrt{x^2 + y^2}

The Attempt at a Solution



Firstly I calculate the length of v and find an expression for the length of u:

||u||=\sqrt{1 + k^2}

||v||=\sqrt{3^2 + 4^2}
||v||=5

Then I find an expression for the dot product:

u{\bullet}v=3+4k

I plug my expressions for the dot product and lengths into the definition of the dot product, and set \theta to \pi/3, giving me:

3+4k=5cos{(\pi/3)}\sqrt{1+k^2}

as cos{(\pi/3)}=1/2, I can substitute cos{(\pi/3)} in my equation for 1/2, giving:

3+4k=5/2\sqrt{1+k^2}

I rearrange and expand brackets to get:

6+8k=5\sqrt{1+k^2}

I then square both sides to get rid of the square root, then expand brackets:

(6+8k)^2=25+25k^2
36 + 96k + 64k^2=25+25k^2

I move everything to one side:

39k^2+96k+11=0

Using the quadratic formula, I get the answer that k= -2.341058209 or -0.1204802515. Clearly one of these is wrong (or both), as there can't be two angles in the same quadrant that make an angle of \pi/3 with a vector. But both of these values do satisfy the equation for the dot product of these two angles when the angle between them is \pi/3. Have I done something wrong?

PS: I wasn't sure if this was the right forum, as both maths forums seem to be calculus-orientated. Sorry if it's not in the right place!
 
Physics news on Phys.org
hi phosgene! :smile:
phosgene said:
Using the quadratic formula, I get the answer that k= -2.341058209 or -0.1204802515. Clearly one of these is wrong (or both), as there can't be two angles in the same quadrant that make an angle of \pi/3 with a vector. But both of these values do satisfy the equation for the dot product of these two angles when the angle between them is \pi/3. Have I done something wrong?

no, you did everything fine :smile:

however, when you squared both sides (which was correct), you automatically introduced an extra solution (for cos = -1/2, ie 2π/3), and you now need to check which of your two solutions is for π/3 ! :wink:

(ie just check that the dot-product is positive)
 


Thanks:) I just found out using google that the dot product is negative if the angle between the vectors is greater than 90 degrees. I had no idea that this was the case (it wasn't mentioned in the lectures). So it's crystal clear now. Thanks again!
 
Question: A clock's minute hand has length 4 and its hour hand has length 3. What is the distance between the tips at the moment when it is increasing most rapidly?(Putnam Exam Question) Answer: Making assumption that both the hands moves at constant angular velocities, the answer is ## \sqrt{7} .## But don't you think this assumption is somewhat doubtful and wrong?

Similar threads

  • · Replies 5 ·
Replies
5
Views
2K
  • · Replies 7 ·
Replies
7
Views
3K
  • · Replies 4 ·
Replies
4
Views
2K
  • · Replies 22 ·
Replies
22
Views
977
Replies
9
Views
2K
  • · Replies 3 ·
Replies
3
Views
2K
Replies
1
Views
1K
Replies
6
Views
2K
  • · Replies 4 ·
Replies
4
Views
2K
  • · Replies 3 ·
Replies
3
Views
2K